Do we know the consistency strength of the Singular Cardinal Hypothesis failing on an uncountable cofinality?Disjoint stationary sets that reflectConsistency strength of the failure of Shelah's Strong Hypothesis (SSH)Prevalent singular cardinals hypothesisSlim Kurepa tree at a singular strong limit cardinal of uncountable cofinalitystationary tower forcingVery weak square and good pointsThe cofinality of the poset $[kappa]^<kappa$ for a singular cardinal $kappa$What are some good lower bounds on the consistency of the failure of the PCF conjecture?

Do we know the consistency strength of the Singular Cardinal Hypothesis failing on an uncountable cofinality?


Disjoint stationary sets that reflectConsistency strength of the failure of Shelah's Strong Hypothesis (SSH)Prevalent singular cardinals hypothesisSlim Kurepa tree at a singular strong limit cardinal of uncountable cofinalitystationary tower forcingVery weak square and good pointsThe cofinality of the poset $[kappa]^<kappa$ for a singular cardinal $kappa$What are some good lower bounds on the consistency of the failure of the PCF conjecture?













13












$begingroup$


Suppose that $kappa$ is a strong limit cardinal. The singular cardinal hypothesis states $2^kappa=kappa^+$. We know that the failure of SCH requires large cardinals, and in fact is equiconsistent with a measurable cardinal $kappa$ satisfying $o(kappa)=kappa^++$.



But this failure is at $aleph_omega$. Suppose we wanted more.



Suppose that we wanted the failure to happen on a couple isolated points. Well, it's not hard to redo the standard constructions and get just that. But what happens when we have limit points?



Even more, by Silver's theorem if SCH fails at $kappa>operatornamecf(kappa)>omega$, then there is a stationary subset of $kappa$ where SCH failed.




What would be the consistency strength when $kappa$ is a singular limit of singular cardinals, and SCH fails cofinally below $kappa$? What if we require $kappa$ to be of uncountable cofinality?




As a side question, what if $kappa$, with uncountable cofinality, does satisfy SCH, but an unbounded subset (which has to be non-stationary, of course) of it does not?










share|cite|improve this question









$endgroup$













  • $begingroup$
    Could you remind the set-theorist dilettantes what $o(kappa)$ stands for?
    $endgroup$
    – Gro-Tsen
    Apr 15 at 10:27






  • 3




    $begingroup$
    @Gro-Tsen: It's the Mitchell order. In some sense, it tells you how large is a measurable. $o(kappa)=1$ means just being a measurable, but $o(kappa)=2$ means that there is a normal measure which concentrates on those $o(lambda)=1$. Which in turn means that the ultrapower by that measure preserves the measurability of $kappa$ itself. You can read more here: cantorsattic.info/Mitchell_rank
    $endgroup$
    – Asaf Karagila
    Apr 15 at 11:11















13












$begingroup$


Suppose that $kappa$ is a strong limit cardinal. The singular cardinal hypothesis states $2^kappa=kappa^+$. We know that the failure of SCH requires large cardinals, and in fact is equiconsistent with a measurable cardinal $kappa$ satisfying $o(kappa)=kappa^++$.



But this failure is at $aleph_omega$. Suppose we wanted more.



Suppose that we wanted the failure to happen on a couple isolated points. Well, it's not hard to redo the standard constructions and get just that. But what happens when we have limit points?



Even more, by Silver's theorem if SCH fails at $kappa>operatornamecf(kappa)>omega$, then there is a stationary subset of $kappa$ where SCH failed.




What would be the consistency strength when $kappa$ is a singular limit of singular cardinals, and SCH fails cofinally below $kappa$? What if we require $kappa$ to be of uncountable cofinality?




As a side question, what if $kappa$, with uncountable cofinality, does satisfy SCH, but an unbounded subset (which has to be non-stationary, of course) of it does not?










share|cite|improve this question









$endgroup$













  • $begingroup$
    Could you remind the set-theorist dilettantes what $o(kappa)$ stands for?
    $endgroup$
    – Gro-Tsen
    Apr 15 at 10:27






  • 3




    $begingroup$
    @Gro-Tsen: It's the Mitchell order. In some sense, it tells you how large is a measurable. $o(kappa)=1$ means just being a measurable, but $o(kappa)=2$ means that there is a normal measure which concentrates on those $o(lambda)=1$. Which in turn means that the ultrapower by that measure preserves the measurability of $kappa$ itself. You can read more here: cantorsattic.info/Mitchell_rank
    $endgroup$
    – Asaf Karagila
    Apr 15 at 11:11













13












13








13


1



$begingroup$


Suppose that $kappa$ is a strong limit cardinal. The singular cardinal hypothesis states $2^kappa=kappa^+$. We know that the failure of SCH requires large cardinals, and in fact is equiconsistent with a measurable cardinal $kappa$ satisfying $o(kappa)=kappa^++$.



But this failure is at $aleph_omega$. Suppose we wanted more.



Suppose that we wanted the failure to happen on a couple isolated points. Well, it's not hard to redo the standard constructions and get just that. But what happens when we have limit points?



Even more, by Silver's theorem if SCH fails at $kappa>operatornamecf(kappa)>omega$, then there is a stationary subset of $kappa$ where SCH failed.




What would be the consistency strength when $kappa$ is a singular limit of singular cardinals, and SCH fails cofinally below $kappa$? What if we require $kappa$ to be of uncountable cofinality?




As a side question, what if $kappa$, with uncountable cofinality, does satisfy SCH, but an unbounded subset (which has to be non-stationary, of course) of it does not?










share|cite|improve this question









$endgroup$




Suppose that $kappa$ is a strong limit cardinal. The singular cardinal hypothesis states $2^kappa=kappa^+$. We know that the failure of SCH requires large cardinals, and in fact is equiconsistent with a measurable cardinal $kappa$ satisfying $o(kappa)=kappa^++$.



But this failure is at $aleph_omega$. Suppose we wanted more.



Suppose that we wanted the failure to happen on a couple isolated points. Well, it's not hard to redo the standard constructions and get just that. But what happens when we have limit points?



Even more, by Silver's theorem if SCH fails at $kappa>operatornamecf(kappa)>omega$, then there is a stationary subset of $kappa$ where SCH failed.




What would be the consistency strength when $kappa$ is a singular limit of singular cardinals, and SCH fails cofinally below $kappa$? What if we require $kappa$ to be of uncountable cofinality?




As a side question, what if $kappa$, with uncountable cofinality, does satisfy SCH, but an unbounded subset (which has to be non-stationary, of course) of it does not?







set-theory lo.logic large-cardinals






share|cite|improve this question













share|cite|improve this question











share|cite|improve this question




share|cite|improve this question










asked Apr 15 at 8:16









Asaf KaragilaAsaf Karagila

22.4k6 gold badges83 silver badges192 bronze badges




22.4k6 gold badges83 silver badges192 bronze badges














  • $begingroup$
    Could you remind the set-theorist dilettantes what $o(kappa)$ stands for?
    $endgroup$
    – Gro-Tsen
    Apr 15 at 10:27






  • 3




    $begingroup$
    @Gro-Tsen: It's the Mitchell order. In some sense, it tells you how large is a measurable. $o(kappa)=1$ means just being a measurable, but $o(kappa)=2$ means that there is a normal measure which concentrates on those $o(lambda)=1$. Which in turn means that the ultrapower by that measure preserves the measurability of $kappa$ itself. You can read more here: cantorsattic.info/Mitchell_rank
    $endgroup$
    – Asaf Karagila
    Apr 15 at 11:11
















  • $begingroup$
    Could you remind the set-theorist dilettantes what $o(kappa)$ stands for?
    $endgroup$
    – Gro-Tsen
    Apr 15 at 10:27






  • 3




    $begingroup$
    @Gro-Tsen: It's the Mitchell order. In some sense, it tells you how large is a measurable. $o(kappa)=1$ means just being a measurable, but $o(kappa)=2$ means that there is a normal measure which concentrates on those $o(lambda)=1$. Which in turn means that the ultrapower by that measure preserves the measurability of $kappa$ itself. You can read more here: cantorsattic.info/Mitchell_rank
    $endgroup$
    – Asaf Karagila
    Apr 15 at 11:11















$begingroup$
Could you remind the set-theorist dilettantes what $o(kappa)$ stands for?
$endgroup$
– Gro-Tsen
Apr 15 at 10:27




$begingroup$
Could you remind the set-theorist dilettantes what $o(kappa)$ stands for?
$endgroup$
– Gro-Tsen
Apr 15 at 10:27




3




3




$begingroup$
@Gro-Tsen: It's the Mitchell order. In some sense, it tells you how large is a measurable. $o(kappa)=1$ means just being a measurable, but $o(kappa)=2$ means that there is a normal measure which concentrates on those $o(lambda)=1$. Which in turn means that the ultrapower by that measure preserves the measurability of $kappa$ itself. You can read more here: cantorsattic.info/Mitchell_rank
$endgroup$
– Asaf Karagila
Apr 15 at 11:11




$begingroup$
@Gro-Tsen: It's the Mitchell order. In some sense, it tells you how large is a measurable. $o(kappa)=1$ means just being a measurable, but $o(kappa)=2$ means that there is a normal measure which concentrates on those $o(lambda)=1$. Which in turn means that the ultrapower by that measure preserves the measurability of $kappa$ itself. You can read more here: cantorsattic.info/Mitchell_rank
$endgroup$
– Asaf Karagila
Apr 15 at 11:11










1 Answer
1






active

oldest

votes


















9














$begingroup$

Suppose $kappa$ is a singular cardinal and there are $cf(kappa)$-many measurable cardinals $lambda < kappa$ with $o(lambda)=lambda^++$ cofinal in $kappa.$ Then you can perform a Prikry type iteration and get the failure of $SCH$ at cofinally many singular cardinals below $kappa.$



Now suppose we also want for $SCH$ to fail at $kappa$ itself. First let us consider the countable cofinality.



Assume $kappa$ is a measurable cardinal with $o(kappa)=kappa^+++1.$ Then we can get an extension in which $cf(kappa)=omega, 2^kappa=kappa^++$ and for cofinally many singular cardinals $lambda$ below $kappa$, we have $2^lambda=lambda^++$. I don't know if this assumption is really needed or if it can be reduced.



For uncountable cofinality, say $theta$, a measurable cardinal $kappa$ with $o(kappa)=kappa^+++theta$ is sufficient. As then you can first find an extension in which $2^kappa=kappa^++$ and such that in the extension, $o(kappa)=theta$. Then if you perform Magidor forcing for changing cofinality of $kappa$ to $theta,$ you can get a club $C$ of singular cardinals below $kappa$ such that for all $lambda in C, 2^lambda=lambda^++$.



As far as I know, if we require $theta=cf(kappa)> omega_1$, the large cardinal assumption is optimal, but for $theta=omega_1,$ I think it is open if this assumption is optimal.






share|cite|improve this answer











$endgroup$














  • $begingroup$
    Note that you may assume that the sequence does not contain its limit points.
    $endgroup$
    – Mohammad Golshani
    Apr 15 at 10:16











  • $begingroup$
    Okay. I sort of expected that to happen. What about the actual question?
    $endgroup$
    – Asaf Karagila
    Apr 15 at 10:16










  • $begingroup$
    What is the question?
    $endgroup$
    – Mohammad Golshani
    Apr 15 at 10:17






  • 1




    $begingroup$
    I assume you say something like this: If for example $o(kappa)=kappa^+++omega_3+3$, then first we can find a model with $2^kappa=kappa^+++o(kappa)=omega_3+3$. Then we can make $cf(kappa)=omega$ by a cofinal sequence $(kappa_n: n<omega)$ of point with $o(kappa_n)=omega_3+2.$ then we can add an $omega$-sequence cofinal in each of these $kappa_n$'s, say $(kappa_n, m: m<omega)$ consisting of points of $o(kappa_n, m)=omega_3+1.$
    $endgroup$
    – Mohammad Golshani
    Apr 16 at 4:18







  • 1




    $begingroup$
    Repeat once more, and find the sequence $(kappa_n,m,l: l<omega)$, cofinal in $kappa_n, m$ with $o(kappa_n, m, l)=omega_3.$ Now apply Magidor's forcing to change the cofinality of each $kappa_n, m, l$ to $omega_3$.
    $endgroup$
    – Mohammad Golshani
    Apr 16 at 4:20













Your Answer








StackExchange.ready(function()
var channelOptions =
tags: "".split(" "),
id: "504"
;
initTagRenderer("".split(" "), "".split(" "), channelOptions);

StackExchange.using("externalEditor", function()
// Have to fire editor after snippets, if snippets enabled
if (StackExchange.settings.snippets.snippetsEnabled)
StackExchange.using("snippets", function()
createEditor();
);

else
createEditor();

);

function createEditor()
StackExchange.prepareEditor(
heartbeatType: 'answer',
autoActivateHeartbeat: false,
convertImagesToLinks: true,
noModals: true,
showLowRepImageUploadWarning: true,
reputationToPostImages: 10,
bindNavPrevention: true,
postfix: "",
imageUploader:
brandingHtml: "Powered by u003ca class="icon-imgur-white" href="https://imgur.com/"u003eu003c/au003e",
contentPolicyHtml: "User contributions licensed under u003ca href="https://creativecommons.org/licenses/by-sa/4.0/"u003ecc by-sa 4.0 with attribution requiredu003c/au003e u003ca href="https://stackoverflow.com/legal/content-policy"u003e(content policy)u003c/au003e",
allowUrls: true
,
noCode: true, onDemand: true,
discardSelector: ".discard-answer"
,immediatelyShowMarkdownHelp:true
);



);














draft saved

draft discarded
















StackExchange.ready(
function ()
StackExchange.openid.initPostLogin('.new-post-login', 'https%3a%2f%2fmathoverflow.net%2fquestions%2f328102%2fdo-we-know-the-consistency-strength-of-the-singular-cardinal-hypothesis-failing%23new-answer', 'question_page');

);

Post as a guest















Required, but never shown

























1 Answer
1






active

oldest

votes








1 Answer
1






active

oldest

votes









active

oldest

votes






active

oldest

votes









9














$begingroup$

Suppose $kappa$ is a singular cardinal and there are $cf(kappa)$-many measurable cardinals $lambda < kappa$ with $o(lambda)=lambda^++$ cofinal in $kappa.$ Then you can perform a Prikry type iteration and get the failure of $SCH$ at cofinally many singular cardinals below $kappa.$



Now suppose we also want for $SCH$ to fail at $kappa$ itself. First let us consider the countable cofinality.



Assume $kappa$ is a measurable cardinal with $o(kappa)=kappa^+++1.$ Then we can get an extension in which $cf(kappa)=omega, 2^kappa=kappa^++$ and for cofinally many singular cardinals $lambda$ below $kappa$, we have $2^lambda=lambda^++$. I don't know if this assumption is really needed or if it can be reduced.



For uncountable cofinality, say $theta$, a measurable cardinal $kappa$ with $o(kappa)=kappa^+++theta$ is sufficient. As then you can first find an extension in which $2^kappa=kappa^++$ and such that in the extension, $o(kappa)=theta$. Then if you perform Magidor forcing for changing cofinality of $kappa$ to $theta,$ you can get a club $C$ of singular cardinals below $kappa$ such that for all $lambda in C, 2^lambda=lambda^++$.



As far as I know, if we require $theta=cf(kappa)> omega_1$, the large cardinal assumption is optimal, but for $theta=omega_1,$ I think it is open if this assumption is optimal.






share|cite|improve this answer











$endgroup$














  • $begingroup$
    Note that you may assume that the sequence does not contain its limit points.
    $endgroup$
    – Mohammad Golshani
    Apr 15 at 10:16











  • $begingroup$
    Okay. I sort of expected that to happen. What about the actual question?
    $endgroup$
    – Asaf Karagila
    Apr 15 at 10:16










  • $begingroup$
    What is the question?
    $endgroup$
    – Mohammad Golshani
    Apr 15 at 10:17






  • 1




    $begingroup$
    I assume you say something like this: If for example $o(kappa)=kappa^+++omega_3+3$, then first we can find a model with $2^kappa=kappa^+++o(kappa)=omega_3+3$. Then we can make $cf(kappa)=omega$ by a cofinal sequence $(kappa_n: n<omega)$ of point with $o(kappa_n)=omega_3+2.$ then we can add an $omega$-sequence cofinal in each of these $kappa_n$'s, say $(kappa_n, m: m<omega)$ consisting of points of $o(kappa_n, m)=omega_3+1.$
    $endgroup$
    – Mohammad Golshani
    Apr 16 at 4:18







  • 1




    $begingroup$
    Repeat once more, and find the sequence $(kappa_n,m,l: l<omega)$, cofinal in $kappa_n, m$ with $o(kappa_n, m, l)=omega_3.$ Now apply Magidor's forcing to change the cofinality of each $kappa_n, m, l$ to $omega_3$.
    $endgroup$
    – Mohammad Golshani
    Apr 16 at 4:20















9














$begingroup$

Suppose $kappa$ is a singular cardinal and there are $cf(kappa)$-many measurable cardinals $lambda < kappa$ with $o(lambda)=lambda^++$ cofinal in $kappa.$ Then you can perform a Prikry type iteration and get the failure of $SCH$ at cofinally many singular cardinals below $kappa.$



Now suppose we also want for $SCH$ to fail at $kappa$ itself. First let us consider the countable cofinality.



Assume $kappa$ is a measurable cardinal with $o(kappa)=kappa^+++1.$ Then we can get an extension in which $cf(kappa)=omega, 2^kappa=kappa^++$ and for cofinally many singular cardinals $lambda$ below $kappa$, we have $2^lambda=lambda^++$. I don't know if this assumption is really needed or if it can be reduced.



For uncountable cofinality, say $theta$, a measurable cardinal $kappa$ with $o(kappa)=kappa^+++theta$ is sufficient. As then you can first find an extension in which $2^kappa=kappa^++$ and such that in the extension, $o(kappa)=theta$. Then if you perform Magidor forcing for changing cofinality of $kappa$ to $theta,$ you can get a club $C$ of singular cardinals below $kappa$ such that for all $lambda in C, 2^lambda=lambda^++$.



As far as I know, if we require $theta=cf(kappa)> omega_1$, the large cardinal assumption is optimal, but for $theta=omega_1,$ I think it is open if this assumption is optimal.






share|cite|improve this answer











$endgroup$














  • $begingroup$
    Note that you may assume that the sequence does not contain its limit points.
    $endgroup$
    – Mohammad Golshani
    Apr 15 at 10:16











  • $begingroup$
    Okay. I sort of expected that to happen. What about the actual question?
    $endgroup$
    – Asaf Karagila
    Apr 15 at 10:16










  • $begingroup$
    What is the question?
    $endgroup$
    – Mohammad Golshani
    Apr 15 at 10:17






  • 1




    $begingroup$
    I assume you say something like this: If for example $o(kappa)=kappa^+++omega_3+3$, then first we can find a model with $2^kappa=kappa^+++o(kappa)=omega_3+3$. Then we can make $cf(kappa)=omega$ by a cofinal sequence $(kappa_n: n<omega)$ of point with $o(kappa_n)=omega_3+2.$ then we can add an $omega$-sequence cofinal in each of these $kappa_n$'s, say $(kappa_n, m: m<omega)$ consisting of points of $o(kappa_n, m)=omega_3+1.$
    $endgroup$
    – Mohammad Golshani
    Apr 16 at 4:18







  • 1




    $begingroup$
    Repeat once more, and find the sequence $(kappa_n,m,l: l<omega)$, cofinal in $kappa_n, m$ with $o(kappa_n, m, l)=omega_3.$ Now apply Magidor's forcing to change the cofinality of each $kappa_n, m, l$ to $omega_3$.
    $endgroup$
    – Mohammad Golshani
    Apr 16 at 4:20













9














9










9







$begingroup$

Suppose $kappa$ is a singular cardinal and there are $cf(kappa)$-many measurable cardinals $lambda < kappa$ with $o(lambda)=lambda^++$ cofinal in $kappa.$ Then you can perform a Prikry type iteration and get the failure of $SCH$ at cofinally many singular cardinals below $kappa.$



Now suppose we also want for $SCH$ to fail at $kappa$ itself. First let us consider the countable cofinality.



Assume $kappa$ is a measurable cardinal with $o(kappa)=kappa^+++1.$ Then we can get an extension in which $cf(kappa)=omega, 2^kappa=kappa^++$ and for cofinally many singular cardinals $lambda$ below $kappa$, we have $2^lambda=lambda^++$. I don't know if this assumption is really needed or if it can be reduced.



For uncountable cofinality, say $theta$, a measurable cardinal $kappa$ with $o(kappa)=kappa^+++theta$ is sufficient. As then you can first find an extension in which $2^kappa=kappa^++$ and such that in the extension, $o(kappa)=theta$. Then if you perform Magidor forcing for changing cofinality of $kappa$ to $theta,$ you can get a club $C$ of singular cardinals below $kappa$ such that for all $lambda in C, 2^lambda=lambda^++$.



As far as I know, if we require $theta=cf(kappa)> omega_1$, the large cardinal assumption is optimal, but for $theta=omega_1,$ I think it is open if this assumption is optimal.






share|cite|improve this answer











$endgroup$



Suppose $kappa$ is a singular cardinal and there are $cf(kappa)$-many measurable cardinals $lambda < kappa$ with $o(lambda)=lambda^++$ cofinal in $kappa.$ Then you can perform a Prikry type iteration and get the failure of $SCH$ at cofinally many singular cardinals below $kappa.$



Now suppose we also want for $SCH$ to fail at $kappa$ itself. First let us consider the countable cofinality.



Assume $kappa$ is a measurable cardinal with $o(kappa)=kappa^+++1.$ Then we can get an extension in which $cf(kappa)=omega, 2^kappa=kappa^++$ and for cofinally many singular cardinals $lambda$ below $kappa$, we have $2^lambda=lambda^++$. I don't know if this assumption is really needed or if it can be reduced.



For uncountable cofinality, say $theta$, a measurable cardinal $kappa$ with $o(kappa)=kappa^+++theta$ is sufficient. As then you can first find an extension in which $2^kappa=kappa^++$ and such that in the extension, $o(kappa)=theta$. Then if you perform Magidor forcing for changing cofinality of $kappa$ to $theta,$ you can get a club $C$ of singular cardinals below $kappa$ such that for all $lambda in C, 2^lambda=lambda^++$.



As far as I know, if we require $theta=cf(kappa)> omega_1$, the large cardinal assumption is optimal, but for $theta=omega_1,$ I think it is open if this assumption is optimal.







share|cite|improve this answer














share|cite|improve this answer



share|cite|improve this answer








edited Apr 15 at 10:31

























answered Apr 15 at 10:09









Mohammad GolshaniMohammad Golshani

20.1k2 gold badges68 silver badges155 bronze badges




20.1k2 gold badges68 silver badges155 bronze badges














  • $begingroup$
    Note that you may assume that the sequence does not contain its limit points.
    $endgroup$
    – Mohammad Golshani
    Apr 15 at 10:16











  • $begingroup$
    Okay. I sort of expected that to happen. What about the actual question?
    $endgroup$
    – Asaf Karagila
    Apr 15 at 10:16










  • $begingroup$
    What is the question?
    $endgroup$
    – Mohammad Golshani
    Apr 15 at 10:17






  • 1




    $begingroup$
    I assume you say something like this: If for example $o(kappa)=kappa^+++omega_3+3$, then first we can find a model with $2^kappa=kappa^+++o(kappa)=omega_3+3$. Then we can make $cf(kappa)=omega$ by a cofinal sequence $(kappa_n: n<omega)$ of point with $o(kappa_n)=omega_3+2.$ then we can add an $omega$-sequence cofinal in each of these $kappa_n$'s, say $(kappa_n, m: m<omega)$ consisting of points of $o(kappa_n, m)=omega_3+1.$
    $endgroup$
    – Mohammad Golshani
    Apr 16 at 4:18







  • 1




    $begingroup$
    Repeat once more, and find the sequence $(kappa_n,m,l: l<omega)$, cofinal in $kappa_n, m$ with $o(kappa_n, m, l)=omega_3.$ Now apply Magidor's forcing to change the cofinality of each $kappa_n, m, l$ to $omega_3$.
    $endgroup$
    – Mohammad Golshani
    Apr 16 at 4:20
















  • $begingroup$
    Note that you may assume that the sequence does not contain its limit points.
    $endgroup$
    – Mohammad Golshani
    Apr 15 at 10:16











  • $begingroup$
    Okay. I sort of expected that to happen. What about the actual question?
    $endgroup$
    – Asaf Karagila
    Apr 15 at 10:16










  • $begingroup$
    What is the question?
    $endgroup$
    – Mohammad Golshani
    Apr 15 at 10:17






  • 1




    $begingroup$
    I assume you say something like this: If for example $o(kappa)=kappa^+++omega_3+3$, then first we can find a model with $2^kappa=kappa^+++o(kappa)=omega_3+3$. Then we can make $cf(kappa)=omega$ by a cofinal sequence $(kappa_n: n<omega)$ of point with $o(kappa_n)=omega_3+2.$ then we can add an $omega$-sequence cofinal in each of these $kappa_n$'s, say $(kappa_n, m: m<omega)$ consisting of points of $o(kappa_n, m)=omega_3+1.$
    $endgroup$
    – Mohammad Golshani
    Apr 16 at 4:18







  • 1




    $begingroup$
    Repeat once more, and find the sequence $(kappa_n,m,l: l<omega)$, cofinal in $kappa_n, m$ with $o(kappa_n, m, l)=omega_3.$ Now apply Magidor's forcing to change the cofinality of each $kappa_n, m, l$ to $omega_3$.
    $endgroup$
    – Mohammad Golshani
    Apr 16 at 4:20















$begingroup$
Note that you may assume that the sequence does not contain its limit points.
$endgroup$
– Mohammad Golshani
Apr 15 at 10:16





$begingroup$
Note that you may assume that the sequence does not contain its limit points.
$endgroup$
– Mohammad Golshani
Apr 15 at 10:16













$begingroup$
Okay. I sort of expected that to happen. What about the actual question?
$endgroup$
– Asaf Karagila
Apr 15 at 10:16




$begingroup$
Okay. I sort of expected that to happen. What about the actual question?
$endgroup$
– Asaf Karagila
Apr 15 at 10:16












$begingroup$
What is the question?
$endgroup$
– Mohammad Golshani
Apr 15 at 10:17




$begingroup$
What is the question?
$endgroup$
– Mohammad Golshani
Apr 15 at 10:17




1




1




$begingroup$
I assume you say something like this: If for example $o(kappa)=kappa^+++omega_3+3$, then first we can find a model with $2^kappa=kappa^+++o(kappa)=omega_3+3$. Then we can make $cf(kappa)=omega$ by a cofinal sequence $(kappa_n: n<omega)$ of point with $o(kappa_n)=omega_3+2.$ then we can add an $omega$-sequence cofinal in each of these $kappa_n$'s, say $(kappa_n, m: m<omega)$ consisting of points of $o(kappa_n, m)=omega_3+1.$
$endgroup$
– Mohammad Golshani
Apr 16 at 4:18





$begingroup$
I assume you say something like this: If for example $o(kappa)=kappa^+++omega_3+3$, then first we can find a model with $2^kappa=kappa^+++o(kappa)=omega_3+3$. Then we can make $cf(kappa)=omega$ by a cofinal sequence $(kappa_n: n<omega)$ of point with $o(kappa_n)=omega_3+2.$ then we can add an $omega$-sequence cofinal in each of these $kappa_n$'s, say $(kappa_n, m: m<omega)$ consisting of points of $o(kappa_n, m)=omega_3+1.$
$endgroup$
– Mohammad Golshani
Apr 16 at 4:18





1




1




$begingroup$
Repeat once more, and find the sequence $(kappa_n,m,l: l<omega)$, cofinal in $kappa_n, m$ with $o(kappa_n, m, l)=omega_3.$ Now apply Magidor's forcing to change the cofinality of each $kappa_n, m, l$ to $omega_3$.
$endgroup$
– Mohammad Golshani
Apr 16 at 4:20




$begingroup$
Repeat once more, and find the sequence $(kappa_n,m,l: l<omega)$, cofinal in $kappa_n, m$ with $o(kappa_n, m, l)=omega_3.$ Now apply Magidor's forcing to change the cofinality of each $kappa_n, m, l$ to $omega_3$.
$endgroup$
– Mohammad Golshani
Apr 16 at 4:20


















draft saved

draft discarded















































Thanks for contributing an answer to MathOverflow!


  • Please be sure to answer the question. Provide details and share your research!

But avoid


  • Asking for help, clarification, or responding to other answers.

  • Making statements based on opinion; back them up with references or personal experience.

Use MathJax to format equations. MathJax reference.


To learn more, see our tips on writing great answers.




draft saved


draft discarded














StackExchange.ready(
function ()
StackExchange.openid.initPostLogin('.new-post-login', 'https%3a%2f%2fmathoverflow.net%2fquestions%2f328102%2fdo-we-know-the-consistency-strength-of-the-singular-cardinal-hypothesis-failing%23new-answer', 'question_page');

);

Post as a guest















Required, but never shown





















































Required, but never shown














Required, but never shown












Required, but never shown







Required, but never shown

































Required, but never shown














Required, but never shown












Required, but never shown







Required, but never shown







Popular posts from this blog

Tamil (spriik) Luke uk diar | Nawigatjuun

Align equal signs while including text over equalitiesAMS align: left aligned text/math plus multicolumn alignmentMultiple alignmentsAligning equations in multiple placesNumbering and aligning an equation with multiple columnsHow to align one equation with another multline equationUsing \ in environments inside the begintabularxNumber equations and preserving alignment of equal signsHow can I align equations to the left and to the right?Double equation alignment problem within align enviromentAligned within align: Why are they right-aligned?

Where does the image of a data connector as a sharp metal spike originate from?Where does the concept of infected people turning into zombies only after death originate from?Where does the motif of a reanimated human head originate?Where did the notion that Dragons could speak originate?Where does the archetypal image of the 'Grey' alien come from?Where did the suffix '-Man' originate?Where does the notion of being injured or killed by an illusion originate?Where did the term “sophont” originate?Where does the trope of magic spells being driven by advanced technology originate from?Where did the term “the living impaired” originate?